Jacobian Transformation - new domain of integration

Click For Summary
The discussion focuses on using the Jacobian transformation to compute a double integral over a specified domain. The transformation defined is s = x + y and t = x - y, with the integration domain D being the first quadrant triangle bounded by x + y ≤ 1. The main challenge is determining the new limits for t, which are stated in the book as -s ≤ t ≤ s. Participants suggest sketching the region and the curves of constant s and t to visualize the boundaries, confirming that the limits for t are derived from the geometry of the transformed region. Understanding these limits is crucial for correctly setting up the integral in the new variables.
IniquiTrance
Messages
185
Reaction score
0

Homework Statement



I need to compute the following using the Jacobian:

\int\int_D \frac{x-y}{x+y} dxdy

Where D = \left\{(x,y):x\geq 0, y\geq 0, x+y \leq 1\right\}

Homework Equations


The Attempt at a Solution



I've made the transformation:

s=x+y \qquad t = x-y

My problem is finding the new domain of integration. I can see that 0\leq s \leq 1.

Yet for t, my book says it should be -s\leq t \leq s, but I cannot see why this should be.

Any help is much appreciated.
 
Physics news on Phys.org
Let r = x e_1 + y e_2 and r' = s e_1 + t e_2 with f(r) = r' and the coordinates s,t as you defined them.

Now, let the boundary of the domain of integration be c(\tau) for some scalar variable tau. There's more than one way to do this, but you can choose

c(\tau) = \begin{cases}<br /> \tau e_1 &amp; (0 &lt; \tau \leq 1) \\<br /> e_1 + (e_2 - e_1)(\tau - 1) &amp; (1 &lt; \tau \leq 2) \\<br /> e_2 + (-e_2)(\tau - 2) &amp; (2 &lt; \tau \leq 3)<br /> \end{cases}

Now, the Jacobian is a linear operator on a vector:
\underline f(e_1) = e_1 + e_2 \\<br /> \underline f(e_2) = e_1 - e_2

The boundary of the domain of integration goes as c(\tau) \to c&#039;(\tau) = \underline f[c(\tau)]. The result of applying the Jacobian operator to c above is

c&#039;(\tau) = \begin{cases}<br /> (e_1 + e_2)\tau &amp; (0 &lt; \tau \leq 1) \\<br /> (e_1+e_2) - 2 e_2 (\tau - 1) &amp; (1 &lt; \tau \leq 2) \\<br /> (e_1 - e_2) + (e_2 - e_1)(\tau - 2) &amp; (2 &lt; \tau \leq 3)<br /> \end{cases}

Drawn up, this is a diagonal from the origin to (s,t) = (1,1), down to (1,-1), and then back to the origin. That's why they say s \geq t \geq -s.
 
omg, TLDR Muphrid!

IniquiTrance said:
...Where D = \left\{(x,y):x\geq 0, y\geq 0, x+y \leq 1\right\}

Homework Equations


The Attempt at a Solution



I've made the transformation:

s=x+y \qquad t = x-y

My problem is finding the new domain of integration. I can see that 0\leq s \leq 1.

Yet for t, my book says it should be -s\leq t \leq s, but I cannot see why this should be.

Any help is much appreciated.

First, find where the boundaries are mapped to, you have x=0, y=0 and x+y=1. These are mapped to x=(s+t)/2=0, y=(s-t)/2=0, and x+y=s=1. Draw these lines as the boundary in the new s,t-space. (Two diagonals through the origin and the vertical line s=1.) If you want to do ∫∫f\ dtds, then for each s value in [0,1] (the projection of the region), t should go from the lower diagonal to the upper diagonal, that is, from t=-s to t=s.
 
IniquiTrance said:

Homework Statement



I need to compute the following using the Jacobian:

\int\int_D \frac{x-y}{x+y} dxdy

Where D = \left\{(x,y):x\geq 0, y\geq 0, x+y \leq 1\right\}

Homework Equations


The Attempt at a Solution



I've made the transformation:

s=x+y \qquad t = x-y

My problem is finding the new domain of integration. I can see that 0\leq s \leq 1.

Yet for t, my book says it should be -s\leq t \leq s, but I cannot see why this should be.

Any help is much appreciated.
First, you should sketch the region in the xy-plane. Next, draw in what curves of constant s and constant t look like on the xy-plane. For example, s=0 corresponds to the line x+y=0; that is, the t-axis is the line y=-x. Just from the sketch, it should be pretty clear what the limits are.
 
Question: A clock's minute hand has length 4 and its hour hand has length 3. What is the distance between the tips at the moment when it is increasing most rapidly?(Putnam Exam Question) Answer: Making assumption that both the hands moves at constant angular velocities, the answer is ## \sqrt{7} .## But don't you think this assumption is somewhat doubtful and wrong?

Similar threads

Replies
7
Views
1K
  • · Replies 105 ·
4
Replies
105
Views
6K
Replies
2
Views
2K
Replies
8
Views
2K
Replies
4
Views
2K
  • · Replies 27 ·
Replies
27
Views
3K
  • · Replies 2 ·
Replies
2
Views
2K
  • · Replies 4 ·
Replies
4
Views
2K
Replies
11
Views
2K
  • · Replies 6 ·
Replies
6
Views
2K